Is the Theory of Magnetism and Relativity Flawed?

In summary, the force on a negatively charged particle from neutral current carrying wire is purely magnetostatic in one frame of reference and purely electrostatic in the other frame of reference using the length contraction. However, taking into account the Magnetic fields, the direction of each B field around each wire determines the direction of the force (in the lab frame) on a test charge or upon ANOTHER CURRENT A CARRYING WIRE.
  • #1
nassboy
39
0
I've seen in feynman and on the internet a derivation that the force on a negatively charged particle from neutral current carrying wire can be shown to be purely magnetostatic in one frame of reference and purely electrostatic in the other frame of reference using the length contraction. The contracted wire has a net positive charge, and therefore attracts the negatively charged particle.

I've tried to extend this idea to two current carrying wires(the direction of the current the same in both wires)...but it seems that they both would have positive charge and repel when they should attract.

What am I doing wrong?
 
Physics news on Phys.org
  • #2
nassboy said:
I've seen in feynman and on the internet a derivation that the force on a negatively charged particle from neutral current carrying wire can be shown to be purely magnetostatic in one frame of reference and purely electrostatic in the other frame of reference using the length contraction.

Hi nassboy! :smile:

That doesn't look right …

E2 - B2 is invariant (independent of the frame of reference), so you can't have the same field with just E or just B in different frames. :confused:
 
  • #5
nassboy said:
I've seen in feynman and on the internet a derivation that the force on a negatively charged particle from neutral current carrying wire can be shown to be purely magnetostatic in one frame of reference and purely electrostatic in the other frame of reference using the length contraction. The contracted wire has a net positive charge, and therefore attracts the negatively charged particle.

I've tried to extend this idea to two current carrying wires(the direction of the current the same in both wires)...but it seems that they both would have positive charge and repel when they should attract.

What am I doing wrong?

Nassboy...
You are failing to take into account the Magnetic fields, in particular the DIRECTION of each B field around each wire.This is the same mistake 'meemoe_uk' was making in a previous thread, (except he would never admit it).

Remember the purpose of that exercise is to derive the MAGNETIC FIELD around the wire...

The DIRECTION of that field around each wire DETERMINES the direction of the force (in the lab frame) on a test charge or upon ANOTHER CURRENT A CARRYING WIRE according to the Lorentz force equation (below)...

F = qv X B (for point charges) (B = mag.field; q = test charge with velocity v outside the wire)

F = Integral ( IL X B) ...( for wires) (where L = length of wire, I = current)
(Your link doesn't point that out very well).
Remember these are the forces YOU as the observer sees in the lab frame.

(Actually, the full Lorentz equaton is F = qE + (qv X B)...but the electric field E is zero in the lab frame since the each wire is electrically neutral in labe frame).

And remember its the 'right hand rule' that determine the direction of the force.
See here for a little more: http://hyperphysics.phy-astr.gsu.edu/hbase/magnetic/magfor.html

Creator

P.S. the link you cited is not a very great explanation since it arrived at the "magnetic force" (eqn. # 3) in somewhat non traditional form...and without the Lorentz force eqn. you cannot see in which direction that force is acting.

However, you will see that the mag. force eqn. (equation # 3 in your link) is the same as my equation above because the term in the parenthesis is simply equal to B...IOW, simply substitute B = uI / 2(pi)R (Ampere's law) for the term in parenthesis and you will recover Lorentz force equation I gave above.)

see:http://hyperphysics.phy-astr.gsu.edu/HBASE/magnetic/magcur.html
 
Last edited:
  • #6
nassboy said:
I've seen in feynman and on the internet a derivation that the force on a negatively charged particle from neutral current carrying wire can be shown to be purely magnetostatic in one frame of reference and purely electrostatic in the other frame of reference using the length contraction. The contracted wire has a net positive charge, and therefore attracts the negatively charged particle.

I've tried to extend this idea to two current carrying wires(the direction of the current the same in both wires)...but it seems that they both would have positive charge and repel when they should attract.

What am I doing wrong?
Let's put the same current in each wire so the mean electron drift velocities are equal. Each moving electron in wire 2 sees a net positive charge in wire 1--this is the case that you say Feynman considers--so the electrons are attracted towards the other wire. The positive copper ions in wire 2 are at rest, however. They see an electrically neutral wire 1, and feel no force. The system is symmetric, so everything we said about wire 2 applies also to wire 1. The mutual force is attractive.

Creator: you've missed the point of this thread. The OP's link demonstrates the relativistic unity of electric and magnetic forces.
 
  • #7
I still don't see why both wires aren't length contracted and therefore have a positive charge...

Only one wire is moving and not both?
 
  • #8
tiny-tim said:
nassboy said:
I've seen in feynman and on the internet a derivation that the force on a negatively charged particle from neutral current carrying wire can be shown to be purely magnetostatic in one frame of reference and purely electrostatic in the other frame of reference using the length contraction.
Hi nassboy! :smile:

That doesn't look right …

E2 - B2 is invariant (independent of the frame of reference), so you can't have the same field with just E or just B in different frames. :confused:

It is possible that [itex]\boldsymbol{E}=0[/itex] and [itex]\boldsymbol{B}\neq 0[/itex] in some frame, so that a particle feels only a magnetic force, and [itex]\boldsymbol{E}\neq 0[/itex] and [itex]\boldsymbol{B}\neq 0[/itex] and [itex]\boldsymbol{v}=0[/itex] in some other frame, so that a particle feels only an electric force.
 
  • #9
nassboy said:
I still don't see why both wires aren't length contracted and therefore have a positive charge...

Only one wire is moving and not both?
The wires aren't moving, only the electrons in the wires.
 
  • #10
nassboy said:
I've seen in feynman and on the internet a derivation that the force on a negatively charged particle from neutral current carrying wire can be shown to be purely magnetostatic in one frame of reference and purely electrostatic in the other frame of reference using the length contraction. The contracted wire has a net positive charge, and therefore attracts the negatively charged particle.

I've tried to extend this idea to two current carrying wires(the direction of the current the same in both wires)...but it seems that they both would have positive charge and repel when they should attract.

What am I doing wrong?

If they both have positive charge they should have repulsive electrostatic force. But the attractive magnetic force due to moving proton will be more. Thus the total force on wire will be attractive. U failed to see the magnetic force.
 
Last edited:
  • #11
lovetruth said:
There is nothing wrong with your understanding. The problem is in the relativity theory and the eroneous derivation by feynmann. I kno i am challenging Feynman and einstein authority but, please read my explanation below for what is wrong in both the theory and derivation. I will only consider the charges(proton and electron) in the wire.
:rolleyes:

So far so good for the magnetostatic case. The real error is done in electrostatic case.

For the electrostatic case,
the proton are at rest so their length is more in this frame than in the magnetostatic frame. The proton density is reduced in the electrostatic case. This increase in length of proton in electrostatic case is ignored in the derivation. The protons length was contracted when the protons were moving in the magnetostatic case. If the protons come at rest in electrostatic frame, they must come to their orignal length which is greater than in magnetostatic frame(in which proton length contraction occur).
electron length will contract and increase electron density(which is correctly shown in the derivation), but proton length expansion is ignored in derivation.
Now u kno the error in derivation.
Even after accounting the error done in derivation, relativity theory gives different result. thus, the theory must be wrong.
Nonsense. Length contraction is most definitely considered.
 

1. What is the relationship between magnetism and relativity?

Magnetism and relativity are two separate concepts in physics, but they are closely related. According to Einstein's theory of special relativity, electric and magnetic fields are two sides of the same coin. When an electrically charged particle moves, it generates an electric field, but due to relativity, the moving particle also experiences a magnetic force. This phenomenon is known as electromagnetism.

2. How does magnetism affect time and space?

According to Einstein's theory of general relativity, the presence of a strong magnetic field can distort space and time. This is known as the magnetic field's curvature, and it is similar to how gravity warps space and time. The effect is small, but it has been observed in experiments and plays a crucial role in understanding the behavior of black holes and other extreme objects in the universe.

3. Can magnetism travel at the speed of light?

Yes, magnetism travels at the speed of light. This is because, as mentioned earlier, magnetism is a result of relativity, and according to Einstein's theory, nothing can travel faster than the speed of light. Therefore, any changes in a magnetic field propagate at the speed of light.

4. How does magnetism affect the flow of time?

Magnetism affects the flow of time by creating a phenomenon known as time dilation. According to special relativity, the closer an object moves to the speed of light, the slower time passes for it. In the presence of a strong magnetic field, particles can reach high speeds, and their time is dilated compared to stationary particles, meaning they experience time slower.

5. Can magnetism be explained by relativity alone?

No, magnetism cannot be explained by relativity alone. While relativity plays a crucial role in understanding the relationship between electricity and magnetism, there are other theories, such as quantum mechanics, that are necessary to fully explain the behavior of magnetism at a microscopic level. However, relativity provides a solid foundation for understanding the macroscopic effects of magnetism.

Similar threads

  • Special and General Relativity
Replies
20
Views
939
  • Special and General Relativity
Replies
27
Views
2K
  • Electromagnetism
Replies
17
Views
1K
  • Special and General Relativity
Replies
34
Views
3K
  • Special and General Relativity
Replies
8
Views
1K
  • Special and General Relativity
5
Replies
161
Views
11K
  • Special and General Relativity
Replies
14
Views
1K
  • Special and General Relativity
Replies
3
Views
1K
  • Introductory Physics Homework Help
Replies
14
Views
1K
  • Special and General Relativity
Replies
9
Views
1K
Back
Top